Mathcenter Forum

Mathcenter Forum (https://www.mathcenter.net/forum/index.php)
-   ข้อสอบโอลิมปิก (https://www.mathcenter.net/forum/forumdisplay.php?f=28)
-   -   3rd TMO ณ ม.นเรศวร (https://www.mathcenter.net/forum/showthread.php?t=1304)

R-Tummykung de Lamar 13 พฤษภาคม 2006 18:11

3rd TMO ณ ม.นเรศวร
 
มาแล้วครับ ค่ายสอวน. วันที่ 8 - 12 พฤษภาคม พ.ศ.2549 ณ ม.นเรศวร
หรือที่เรียกกันว่า การแข่งขันคณิตศาสตร์โอลิมปิก ครั้งที่ 3






gools 13 พฤษภาคม 2006 18:22

เห็นข้ออสมการแล้วรู้สึกอยากทำยังไงก็ไม่รู้ :D
\[1+\frac{3}{ab+bc+ca} \geq 1+\frac{9}{(a+b+c)^2}\]
\[1+\frac{9}{(a+b+c)^2} \geq \frac{6}{a+b+c} \iff (\frac{3}{a+b+c}-1)^2 \geq 0\]

gools 13 พฤษภาคม 2006 18:44

ข้อ 3 วันที่ 2 ครับ

ให้ $\omega^3=1$
แทน $x$ ด้วย $\omega$ และ $\omega^2$ ในสมการ
จะได้ว่า
\[\begin{array}{rcl} 2\omega P(1)+Q(1) &=& 0 \ \ \ldots(1) \\
2\omega^2 P(1)+Q(1) &=& 0 \ \ \ldots(2)
\end{array}
\]
$(1)+(2)$ จะได้ $Q(1)-P(1)=0$
ดังนั้น $x-1$ เป็นตัวประกอบของ $Q(x)-P(x)$

R-Tummykung de Lamar 13 พฤษภาคม 2006 18:48

30 นาทีผ่านไป คุณ gools เกือบจะได้หรียญเงินแล้ว (หรือได้แล้วหว่า)
อิอิ :D

gools 13 พฤษภาคม 2006 18:52

เกณฑ์การให้รางวัลเป็นยังไงอ่ะครับ

R-Tummykung de Lamar 13 พฤษภาคม 2006 18:56

อ้างอิง:

ข้อความเดิมของคุณ gools:
เกณฑ์การให้รางวัลเป็นยังไงอ่ะครับ
เทียบจากผู้เข้าร่วมแข่งขันครับ
แบ่งผู้เข้าสอบเป็น 12 ส่วน
ส่วนแรกได้เหรียญทอง (ดีเยี่ยม)
2 ส่วนถัดมาได้เหรียญเงิน (ดีมาก)
3 ส่วนถัดมาได้เหรียญทองแดง (ดี)
อีก 6 ส่วน ได้รับเกียรติบัตรเข้าร่วมการแข่งขัน

แล้วก็ 40 คนแรก ได้ผ่านไปสอบ สสวท. รอบสองโดยไม่ต้องสอบรอบแรก

เหรียญทองจะตัดที่ 28 ครับ ส่วนเหรียญเงินผมไม่แน่ใจ และเหรีญทองแดง รู้สึกจะตัดที่ 7 :yum:

nongtum 13 พฤษภาคม 2006 19:08

ยังไม่ได้ดูทั้งหมด แต่แว้บมาปั่นกระทู้ก่อน :P

สามข้อแรกเสร็จผมกับคุณ Passer-by ไปแล้ว ขอเติมตอบอีกสามข้อก่อนละกัน
5. เมื่อแทน m=1,n=0 จะได้ว่า f(1)-f(0)=1 เพราะ $17|2006$ แต่ $17\not\vert1997$ ดังนั้น f(0)=2006/17=118
8. $s^2=a+b+c+2\sum_{cyc}\sqrt{ab}=9+22=31$ ดังนั้น $s^4-18s^2-8s=(31-18)31-8\sqrt{31}=403-8\sqrt{31}$ (ดูที่ความคิดเห็นของคุณ Passer-by ด้านล่าง)
15. เพราะ 49x50<2549<50x51 ดังนั้นมี n ที่สอดคล้อง 49 ตัว

R-Tummykung de Lamar 13 พฤษภาคม 2006 19:23

ข้อ 8 ของพี่ nongtum ยังไม่ถูกครับ
$s^2=a+b+c+2\sum_{cyc}\sqrt{ab}=9+22=31$
ตรงนี้คับ $\sum_{cyc}\sqrt{ab}\not=\sum_{cyc}{ab}=11$

episer 13 พฤษภาคม 2006 20:42

อ้าว ไม่ใช่ทั้ง 44 คนที่ได้เหรียญจะติดไปหมดเรยเหรอ

ผมนึกว่าเค้าเอา ทั้ง 44 แต่ tummykung บอกว่า 40

Pheeradej 13 พฤษภาคม 2006 20:55

เหรียญทองตัดที่ 27 คับ เพราะเพื่อนผมที่ได้ อันดับสุดท้ายของเหรียญทองได้ 27 คะแนน ท็อปได้ 43 คะแนน และถ้าจำไม่ผิดเหรียญทองแดงตัดที่ 6 คะแนน และเหรียญเงินตัดที่ 15 คะแนน ครับ

nongtum 13 พฤษภาคม 2006 21:35

14. $2549|n^{2545}-2541\ \Rightarrow\ 2549|n^3(n^{2545}-2541)$ เนื่องจาก 2549 เป็นจำนวนเฉพาะ โดย Fermat จะได้
$n^{2548}-2541n^3\equiv1+8n^3\pmod{2549}$ เนื่องจาก $8n^3+1=(2n+1)(4n^2-2n+1)$ และ $4n^2-2n+1=2549$ ไม่มีคำตอบเป็นจำนวนเต็ม ดังนั้นค่า n ที่น้อยที่สุดคือ (2549-1)/2=1274

nongtum 13 พฤษภาคม 2006 21:37

ยังไม่ได้แก้ข้อ 8 แต่มาแปะเพิ่มอีกสี่ข้อครับ :p

4. ลาก PO, AB และให้ AQ=2x จะได้ AP=PB=3x, BQ=4x ให้ r เป็นรัศมีวงกลม ดังนั้น $(4x-r)^2=4x^2+r^2$ หรือ r=3x/2
เนื่องจาก QAC=CBA=OPB ในที่สุดจะได้ $\sin\hat{CAQ}=1/\sqrt5$

6. แทน (0,0) ได้ $f(1)=f(1)+2(f(0))^2$ ดังนั้น $f(0)=0$
แทน (x,0) ได้ $f(\cos x)=\cos x\; f(1)$ แทน cos x ด้วย z จะได้ $f(z)=f(1)z$
แทน $(\pi,0)$ ได้ $f(-1)=-f(1)$
แทน $(\pi/2,\pi/2)$ จะได้ $2(f(1))^2=f(-1)$ ในที่สุดจะได้ว่า f(1)=-1/2 และ $f(\frac{2006}{2549})=-\frac{1003}{2549}$

10. โดย Wilson จะได้ $(29-1)!\equiv28\cdot27!\equiv28\pmod{29}$ ซึ่งหมายถึง $27!\equiv1\pmod{29}$
$27\cdot26!\equiv-2\cdot26\equiv1\pmod{29}$ ดังนั้น $26!\equiv14\pmod{29}$
$2^5\equiv3(29),\ 2^{25}\equiv243\equiv11(29)\ \Rightarrow\ 2^{26}\equiv-7\pmod{29}$
$7^3\equiv343\equiv-5(29),\ 7^6\equiv25\equiv-4(29),\ 7^{24}\equiv256\equiv-5\pmod{29}$ ดังนั้น $7^{26}\equiv(-9)(-5)=45\equiv16\pmod{29}$
$\Rightarrow\ (26!)^{26}\equiv14^{26}\equiv16\cdot(-7)\equiv4\pmod{29}$
ดังนั้นเศษจากการหารที่ต้องการคือ 4+1=5

12. $a_1=710,\ 71\not\vert{a_2},\ 10\not\vert{a_3},\ 2|a_n\ \forall n$ ดังนั้น gcd(...)=2 ตอบ 14 วิธีทำดูด้านล่าง

R-Tummykung de Lamar 13 พฤษภาคม 2006 22:07

ข้อ 12
$a_1=210$ ไม่ใช่ $710$ คับ
ข้อนี้ผมก็ตอบ 2 ไป (เพราะคิดไม่ออก T_T) แต่ไปถามเพื่อนเค้าตอบ 14 อะครับ ไม่ทราบว่า ทำอย่างไรครับ (ผมไม่กล้ากระจาย $a_2,a_3$ อะคับ กลัวเลขเยอะ)

nongtum 13 พฤษภาคม 2006 22:44

เอ้า แก้กันอีกรอบ

12. $a_1=210,\ 3\not\vert{a_2},\ 5\not\vert{a_2},\ 2|a_n\ \forall n$ เราจะแสดงว่า $7|a_n\ \forall n$
$2^3\equiv1\pmod7\ \Rightarrow\ 2^{3n}\equiv1\pmod7\ \Rightarrow\ 4\cdot2^{3n}=2^{3n+2}\equiv2^{3n-1}\equiv4\pmod7$
$3^6\equiv1\pmod7\ \Rightarrow\ 3^{6n}\equiv1\pmod7\ \Rightarrow\ 81\cdot3^{6n}\equiv3^{6n-2}\equiv81\equiv-3\pmod7$
$5^6\equiv1\pmod7\ \Rightarrow\ 5^{6n}\equiv1\pmod7\ \Rightarrow\ 5^{6n-3}\equiv125\equiv-1\pmod7$
รวมเศษที่ได้เป็นอันเสร็จพิธี

passer-by 14 พฤษภาคม 2006 04:25

ตอนที่ 1

7. By Cauchy schwarz's inequality

$ 1=\frac{x}{2}+\frac{x}{2}+\frac{y}{3}+ \frac{y}{3}+ \frac{y}{3}+ \frac{z}{4}+\frac{z}{4}+ \frac{z}{4}+ \frac{z}{4} \leq \sqrt{2x^2+3y^2+4z^2}\sqrt{2(\frac{1}{4})+3(\frac{1}{9})+4(\frac{1}{16})} $

หรือ $ 2x^2+3y^2+4z^2 \geq \frac{12}{13} $

และ สมการเป็นจริง เมื่อ มี l>0 ซึ่ง
x=l/2
y=l/3
z=l/4

แก้สมการ และแทนค่ากลับไป จะได้ x= 6/13

8. ให้ $ t=\sqrt{ab}+\sqrt{bc}+\sqrt{ca} $

จากโจทย์จะได้ $ s^2 = (a+b+c) + 2t = 9+2t $
ขณะเดียวกัน $ t^2= (ab+bc+ca)+ 2\sqrt{abc}(s) = 11 +2s $

กำจัด t ให้หมดไป จะได้ $ s^4-18s^2-8s = -37 $

9.

พิจารณา $ \frac{(n+1)^3}{n(n-1)} - \frac{(n-1)^3}{n(n+1)} = \frac{1}{n}\bigg (\frac{(n+1)^4-(n-1)^4}{n^2-1} \bigg ) = \frac{1}{n}\bigg (\frac{8n(n^2+1)}{n^2-1}\bigg )= 8(1+\frac{2}{n^2-1}) $

ถ้า n= 2548 เทอมในวงเล็บจะเข้าใกล้ 1 ดังนั้น คำตอบข้อนี้ คือ 8

13.
ให้ x แทนจำนวนดังกล่าว

ดังนั้น $ 10^{1862}-1 = 9x $

By Fermat's little theorem $ 10^{58}\equiv 1\pmod {59} $ และ $ 10^{16}\equiv 1\pmod {17} $

เพราะ $1856 = 2\cdot 58 \cdot 16 $ ดังนั้น
$ 10^{1856}\equiv 1 \pmod {59} $ และ $ 10^{1856}\equiv 1 \pmod {17} $

และเพราะ (59,17)=1 ทำให้ $ 10^{1856} \equiv 1 \pmod {59\cdot 17} $

ดังนั้น $ 10^{1862}-1 \equiv 10^6-1 \pmod {59\cdot 17} $
หรือ $ 9x \equiv 10^6-1 \pmod {1003} \equiv (10^3+1)(10^3-1) \pmod {1003} \equiv 9(111)(1001) \pmod {1003} \equiv 9(111)(1003)- 9(111)(2) \pmod {1003} $

Simplify เป็น $ x \equiv -222 \pmod {1003} \equiv 781 \pmod {1003}$

ขณะเดียวกัน $ x \equiv 781 \pmod {2} $ ด้วย และเพราะ (1003,2)=1 ดังนั้น $ x \equiv 781 \pmod {2006} $
คำตอบ คือ 781

18.

แปลงปัญหาเป็น มีวิธีเลือก เลข 10 ตัว จาก {1,2,...,31} กี่วิธีที่ ไม่มี 2 ตัวใดติดกัน

สมมติมีเลข 1 10 ตัว และเลข 0 อีก 21 ตัว

จำนวนวิธีเรียงเลข 0,1 ดังกล่าว โดย 1 แยกกันหมด หรือ $ {22 \choose 10}$จะเทียบเท่ากับคำตอบ ข้อนี้ (โดยให้หลักซ้ายสุดคือเลข 1 นับไปเรื่อยๆ จนถึงหลักขวาสุด คือ เลข 31)

ตอนที่ 2
4.
ข้อนี้เป็นคำถามยอดฮิตใน combinatorics เลยครับ คาดว่าคนตั้งโจทย์คงดัดแปลงมาจาก คำถามที่ว่า ถ้าระบายสี 2 สี บน lattice grid แล้วจะมีสี่เหลี่ยมมุมฉากอย่างน้อย 1 รูป ที่จุดมุม ทาสีเดียวกัน

เนื่องจาก กลุ่มหนึ่งมี 7 คน โดยหลักรังนกพิราบ จะมีอย่างน้อย 4 คนที่เพศเดียวกัน สมมติเป็นชาย
พิจารณาสมาชิกกลุ่มที่เหลืออีก 3 กลุ่มที่มีหมายเลข ตรงกับ 4 คนดังกล่าว (เท่ากับคิดเฉพาะ 12 คนนี้) แล้วแบ่งเป็น 2 กรณี
(1) ถ้า มี 2 คนในกลุ่มใดกลุ่มหนึ่งที่เหลือ มี เพศชาย , proof complete
(2) สำหรับแต่ละกลุ่มที่เหลือ ถ้ามีสมาชิกอย่างมาก 1 คน เป็นชาย หรือเท่ากับว่า อย่างน้อย 3 คนเป็นหญิง ก็จะหาคุณสมบัติที่โจทย์ต้องการได้เช่นกัน สำหรับกรณีเพศหญิง

ข้อนี้อธิบายยากจัง :dry: ถ้าใครไม่เข้าใจ ลองวาดรูปตามหรือดูจาก lattice grid จะง่ายที่สุดครับ

ข้อ 16 ตอบ 72549 หรือเปล่าครับ

p.s. น้อง tummykung check pm ด้วยครับ

nongtum 14 พฤษภาคม 2006 05:37

คุณ Passer-by แก้หา x ผิดหรือเปล่า เพราะจากเงื่อนไขด้านบน ผมได้ $\lambda(\frac12+\frac13+\frac14)=\frac{13}{12}\lambda=\frac{12}{13}$ ซึ่งจะได้ $\lambda=(\frac{12}{13})^2$ และ x=72/169

สุดยอดมากครับ ทั้งคุณ Passer-by น้อง Tummykun และน้อง Gools :great:

passer-by 14 พฤษภาคม 2006 05:46

จากโจทย์ x+y+z =1 นะคร้าบ

[Cb : TkZ] 14 พฤษภาคม 2006 12:16

ตอนที่ 2 ข้อ 5 ยากจัง
คัยทำได้มั่งคับ
แสดง ให้ดูหน่อยซิคับ

R-Tummykung de Lamar 14 พฤษภาคม 2006 12:25

เท่าที่ดูมา คำตอบของพี่ Passer-by ไม่มีปัญหาเลยครับ :great:

ตอนที่ 2 ข้อ 4 (Alternate)
สร้างตารางขนาด $4\times 7$ โดยแต่ละแถวเป็นกลุ่ม แต่ละคอลัมน์เป็นเลขที่
ผมตั้งสมมติฐานครับ ว่า ไม่มีอย่างที่โจทย์ต้องการ นั่นคือ มี อย่างมาก 1 คู่ จากสองแถว ที่อยู่ในคอลัมน์เดียวกัน และเป็นเพศเดียวกัน

โดยหลักรังนกพิราบ ในแต่ละแถวจะต้องมีเพศเดียวกันอย่างน้อย 4 คน ซึ่งเป็นเพศที่มากกว่า (ผมเรียกเพศนี้ว่า "เพศเอก" ของแต่ละแถว)

เนื่องจากเพศเอกมีได้ 2 เพศ (ช/ญ) ดังนั้น 4 แถว โดยหลักรังนกพิราบจะได้ว่า มีเพศเอกเป็นเพศเดียวกัน อย่างน้อย 2 แถว สมมติว่าเพศชาย

เพื่อไม่ให้ขัดแย้งกับสมมติฐาน 2 แถวที่มีเพศเอกป็นเพศชายนั้น จะวางตรงกัน 1 คู่ อีก 3 คน จะต้องไม่ตรงกัน และคนที่เหลือในแถวนั้นต้องเป็นเพศหญิงหมด

แถวที่เหลือ หากมีเพศชายเป็นเพศเอกอีก สมมติฐานจะขัดแย้ง ถ้าเป็นเพศหญิง สมมติฐานก็ขัดแย้งอยู่ดี
เป็นอันจบการพิสูจน์

แหะๆ ดูจะยุ่งยากกว่าของพี่ Passer-by มากทีเดียว แต่ก็พอจะเป็นอีกแนวคิดละกันครับ

Pheeradej 14 พฤษภาคม 2006 13:31

ตอนที่ 2 ข้อ1 คับ
สมมติมีจำนวนนับที่เรียงถัดกัน 3 จำนวน ที่สอดคล้องเงื่อนไข
ให้ A เป็นจำนวนที่ 2 (จำนวนตรงกลาง)
จะได้จำนวนดังกล่าวคือ (A-1)(A)(A+1) = A(A2-1)
เนื่องจาก (A,A2-1) = 1
ดังนั้น A=a2 ,A2-1 = b2 $a,bN
ซึ่งจาก case A2-1 = b2 พบว่าเป็นไปไม่ได้
จึงเกิดข้อขัดแย้งขึ้น จบการพิสูจน์คับ

passer-by 14 พฤษภาคม 2006 15:30

ข้อ 16 ผมคิดแบบ กำปั้นทุบดินมากๆ ไม่รู้ถูกหรือเปล่า
ถ้าพิจารณา แผนภาพ Venn-Euler ของ ABC พบว่ามี 7 บริเวณ
และ เลข 1-2549 ต้องบรรจุในแผนภาพนี้ บริเวณใดบริเวณหนึ่ง ดังนั้นเลข 1 ตัว มีทางเลือก 7 แบบ
สรุปว่ามีวิธีสร้างเซต A,B,C ได้ 72549 วิธี

เท่าที่ดูมาทั้งหมด ผมว่า ข้อ 5 ตอนที่ 2 ยากที่สุดแล้วมั้งครับ (ยากพอๆกับ ข้อ composite function ของ TMO ปีที่แล้วเลย) รอเซียน Number theory มาตอบดีกว่า

แล้วก็ขอถามน้องที่ไปแข่งมาน่ะครับ ว่า คะแนนเต็มแต่ละข้อ เท่าไหร่บ้าง

ตอนนี้เหลือ ข้อ 11,17 (ตอนที่ 1) และข้อ 2,5 (ตอนที่ 2) เซียนๆทั้งหลาย รีบมา post กันนะครับ

warut 14 พฤษภาคม 2006 17:32

ข้อ 5. ตอนที่ 2 มาแนวเดียวกับ IMO 2005 ข้อ 4. (ดูได้ที่ ข้อ 7. Number Theory มาราธอน ครับ) แต่ยากกว่า!

ให้ $p=2549$ ดังนั้น $p$ เป็นจำนวนเฉพาะ
ให้ $m=(p-3)/2$ และ $n=p-2$ จะเห็นว่า $(m,n)=1$ เพราะ $n-2m=1$
ให้สังเกตว่า $$(25\cdot 49)((25\cdot 49)^m +25^n- 2\cdot49^n) $$ $$ =(5\cdot7)^{p-1}+ 49\cdot 25^{p-1} -2\cdot25 \cdot49^{p-1}$$ $$ \equiv1+49-2\cdot25 \equiv 0 \pmod p$$ เราจึงได้ว่า $2549 \mid (25\cdot 49)^m +25^n- 2\cdot49^n$ ตามที่ต้องการครับ :)

nongtum 14 พฤษภาคม 2006 19:20

ตอนที่ 2 ข้อ 2
กำหนด BAQ=x=QPC, AQR=RQC=y, PAQ=z=ACQ, AQB=w ดังนั้นจะได้ว่า
$\displaystyle\frac{\sin y}{RC}=\frac{\sin z}{QR},\quad\frac{\sin y}{AR}
=\frac{\sin z}{QR}
\quad\Rightarrow\quad\frac{AR}{RC}=\frac{\sin z}{\sin x}=:k$
$\displaystyle\frac{\sin z}{PQ}=\frac{\sin (180-w)}{PA}=\frac{\sin w}{PA},
\quad\frac{\sin x}{PQ}=\frac{\sin (2y-w)}{PA}
\quad\Rightarrow\quad\frac{\sin z}{\sin x}=\frac{\sin w}{\sin (2y-w)}$
$\displaystyle\frac{\sin w}{AB}=\frac{\sin x}{QB},\quad\frac{\sin (2y-w)}{BC}
=\frac{\sin z}{QB}
\quad\Rightarrow\quad\frac{AB}{BC}=\frac{\sin w}{\sin (2y-w)}\cdot\frac{\sin z}{\sin x}=k^2$
จบการพิสูจน์ :tired:

gon 14 พฤษภาคม 2006 20:18

ผมทำตอนที่ 1 ข้อ 17 แล้วกัน ตอบ 16 วิธี
ใครลองทำดูแล้วตรวจดูด้วยนะครับ.;)

สมมติให้ A, B, C D, E, F เป็นสมาชิกที่แตกต่างกันของเซต {1, 2, 3, 4, 5, 6}

เมื่อพิจารณาค่า A จะพบว่า A = 1 เท่านั้นที่เป็นไปได้

เมื่อพิจารณาค่า B จะพบว่า B = 2, 3, 4 เท่านั้นที่เป็นไปได้
เพราะถ้า B = 5 แล้วสมาชิกที่เหลือคือ 2, 3, 4, 6 จะต้องมีอย่างน้อย 2 จำนวน (D, E) ที่มีค่ามากกว่า 5 ซึ่งไม่มี ในทำนอง B = 6 ก็เช่นเดียวกัน

จึงมีกรณีที่ต้องพิจารณาทั้งหมด 3 กรณีใหญ่ ๆ
กรณีที่ 1 , B = 2
มีกรณีย่อยที่ต้องพิจารณาอีก 2 กรณีคือ C = 3 หรือ 4 ด้วยเหตุผลเดียวกับที่พิจารณาค่าของ B
กรณีที่ 1.1 , C = 3 : D, E, F จะอยู่ใน {4, 5, 6} ซึ่งสามารถสลับที่ได้ทั้งหมด 3! วิธี

กรณีที่ 1.2 , C = 4
เนื่องจาก 4 ต้องน้อยกว่าทั้ง E และ F ดังนั้น E กับ F จะต้องเป็น 5 หรือ 6 และ D = 3 เท่านั้น ดังนั้น คำตอบในกรณีนี้จะมี 2! วิธี

กรณีที่ 2 , B = 3
มีกรณีย่อยที่ต้องพิจารณาอีก 2 กรณีคือ C = 2 หรือ 4 ด้วยเหตุผลเดียวกับที่พิจารณาค่าของ B
กรณีที่ 2.1 , C = 2 จำนวนคำตอบจะเท่ากับกรณีที่ 1.1 คือ 3! วิธี

กรณีที่ 2.2 , C = 4 จะพบว่ากรณีนี้เป็นไปไม่ได้ เพราะ E กับ F จะต้องเป็น 5 หรือ 6 เท่านั้น ซึ่งทำให้ D = 2

กรณีที่ 3 , B = 4
มีกรณีย่อยที่ต้องพิจารณาอีก 2 กรณีคือ C = 2 หรือ 3 ด้วยเหตุผลเดียวกับที่พิจารณาค่าของ B

กรณีที่ 3.1 , C = 2 จำนวนคำตอบจะเท่ากับกรณีที่ 1.2 คือ 2! วิธี

กรณีที่ 3.2 , C = 3 จะพบว่ากรณีนี้เป็นไปไม่ได้ เพราะ D กับ E จะต้องเป็น 5 หรือ 6 เท่านั้น ซึ่งทำให้ F = 2

ดังนั้นคำตอบทั้งหมดจึงเป็น 3! + 2! + 3! + 2! = 16 วิธี :mad:

nongtum 14 พฤษภาคม 2006 22:06

11. จาก $2006\equiv4\pmod{13},\ 4^{12}\equiv1\pmod{13}$ และ $2^{12}=4^6\equiv1\pmod{13}$ จะได้ว่า
$4^{(6k+1)^2-1}=(4^{12})^{3k^2+k}\equiv1\pmod{13}$ และ $4^{(6k+5)^2-1}=(4^6)^{6k^2+10k+4}\equiv1\pmod{13}$
ดังนั้น $\prod_{i=1}^{2549}\;2006^{{p_i}^2-1}
\equiv4^{3+8}\prod_{i=3}^{2549}\;2006^{{p_i}^2-1}\equiv4^5\cdot1=10\pmod{13}$

edit: ตก 3 ไปได้ไงเนี่ย... :eek:

passer-by 15 พฤษภาคม 2006 01:11

ตอนที่ 2 ข้อ 2 (Alternative solution)

ให้ $ C \hat{A}Q= x \quad A\hat{C}Q= z \quad A\hat{P}Q= \theta_1 \quad Q\hat{P}C= \theta_2 $

จากที่คุณ nongtum ทำไว้ $ \frac{AR}{RC}= \frac{\sin z}{\sin x} $

จากนั้น ใช้ law of sine กับสามเหลี่ยม APQ และ PQC จะได้
$\frac{AQ}{\sin\theta_1}=\frac{PQ}{\sin (\pi-z)} \,\, \text{and} \,\, \frac{QC}{\sin \theta_2}=\frac{PQ}{\sin (\pi-x)} \rightarrow (\frac{\sin \theta_1}{\sin \theta_2})(\frac{QC}{AQ})=\frac{\sin z}{\sin x}=\frac{AR}{RC} \cdots (1) $

ประกอบกับการเทียบพื้นที่ในสามเหลี่ยมย่อยใน APC และ AQC จะได้ความสัมพันธ์

$ \frac{AR}{RC}=\frac{AQ}{QC} \cdots (2) $
$ \frac{AB}{BC}=\frac{\sin\theta_1}{\sin\theta_2} \cdots(3) $

จาก (2) ,(3) แทนใน (1) จะได้สิ่งที่โจทย์ต้องการ

ส่วนข้อ 11 ตอนที่ 1
ผมว่าคุณ nongtum ลืมพิจารณาจำนวนเฉพาะ 3 ไปนะครับ เพราะ 6k1 ไม่ cover เลข 3

และบรรทัดสุดท้าย น่าจะเป็น -1 ที่อยู่หน้าเครื่องหมาย product นะครับ ไม่น่าจะเป็นเลข 4

p.s. สำหรับน้องที่ผ่านรอบนี้แล้ว และอยากเพิ่มศักยภาพให้ตัวเอง ลองไปหาข้อสอบ training olympiad ของจีน มาลองทำดูก็ดีครับ ผมว่าโหดสุดๆแล้ว

nongtum 15 พฤษภาคม 2006 02:02

ขอบคุณสำหรับคำท้วงติงครับ แก้คราวนี้หวังว่าจะไม่มีที่ผิด แต่ถ้ายังเจอหรือมีวิธีทำที่ต่างจากนี้ก็บอกกันได้ครับ

1. ข้อสอบชุดนี้มีข้อที่ต้องใช้ Little Fermat ช่วยตั้งสี่ห้าข้อ ไม่อยากนึกเลยว่าหากต้องไปนั่งสอบด้วยจริงจะทำได้อย่างนี้ไหม อยากรู้เหมือนกันว่าแต่ละข้อมันกี่คะแนน :o
2. เพิ่มลิงค์ TMO 3rd ในหน้ารวมลิงค์ข้อสอบแล้วนะครับ

warut 15 พฤษภาคม 2006 06:00

ข้อ 17. ตอนที่ 1 ผม simplify วิธีคิดของคุณ gon ได้โดยแบ่งเป็น 2 กรณีดังนี้ครับ

กรณีที่ 1 แถวกลางเป็นเลข 2 กับ 3
ดังนั้นแถวกลางจึงเรียงได้ 2! วิธี: 2,3 กับ 3,2
ส่วนแถวล่างซึ่งประกอบด้วย 4, 5, 6 จะเรียงได้ 3! วิธี
ดังนั้นในกรณีที่ 1 นี้จึงต่อตัวได้ทั้งหมด 2!3! วิธี

กรณีที่ 2 แถวกลางเป็นเลข 2 กับ 4
ดังนั้นแถวกลางจึงเรียงได้ 2! วิธี: 2,4 กับ 4,2
ส่วนแถวล่างซึ่งประกอบด้วย 3, 5, 6 จะเรียงได้ 2! วิธี เพราะเราทำได้แค่สลับ 5 กับ 6 ใต้ 4 เท่านั้น
ดังนั้นในกรณีที่ 2 นี้จึงต่อตัวได้ทั้งหมด 2!2! วิธี

รวมต่อตัวได้ 2!3! + 2!2! = 16 วิธีครับ :)

R-Tummykung de Lamar 15 พฤษภาคม 2006 08:43

ตอนแรก (เติมคำตอบ) 18 ข้อ ข้อละ 1 คะแนน รวม 18 คะแนน
ตอนที่สอง (แสดงวิธีทำ) 6 ข้อ ข้อละ 7 คะแนน รวม 42 คะแนน

รวมสองวัน 60 คะแนน

ปล.ข้อ 5 พี่ warut ทำยังไงถึงจะรู้ว่าให้ m,n เท่ากับตัวนั้นอะครับ :D

warut 15 พฤษภาคม 2006 12:47

ใช้การลองผิดลองถูก บวกกับประสบการณ์ที่ได้จากการทำโจทย์ IMO ข้อนั้นครับ

Edit: ลืมบอกไปว่า ค่าของ m, n ที่ผมใช้นั่น ถ้าสลับกันก็ยังใช้ได้ครับ

zzz010307 16 พฤษภาคม 2006 17:17

ข้อสอบโอลิมปิกสอวน. ภาคอัตนัย(แสดงวิธีทำ)
กับข้อสอบคัดเลือกโอลิมปิกสสวท. รอบที่2
อันไหนยากกว่ากันหรือครับ?

Pheeradej 16 พฤษภาคม 2006 22:27

ผมคิดว่าข้อ 2 อาจจะทำได้มากกว่า 5 วิธีอีกนะครับ (รวมวิธีของคุณ passer-by กับคุณ nongtum ผมเห็นมา 4 วิธีแล้ว) ส่วนวิธีของผมตอนสอบผมทำดังนี้
ให้ QAC = x QCA = y
จากสมบัติวงกลมจะได้ PCQ = QAC = x PAQ =QCA = y
จาก law of sine กับ สามเหลี่ยม AQC จะได้
$\frac{AQ}{QC}$ = $\frac{siny}{sinx}$
\$\frac{AB}{BC}$ = $\frac{[PAQ]}{[PCQ]}$ = $\frac{AQsiny}{QCsinx}$ (โดยสูตรพท.สามเหลี่ยม)= $\frac{AQ^{ 2 }}{QC^{ 2 }}$ =$\frac{AR^{ 2 }}{RC^{ 2 }}$ ตามต้องการ
ส่วนอีกวิธีนึงใช้สามเหลี่ยมคล้ายครับ

bell18 18 พฤษภาคม 2006 15:16

ตอบคุณ zzz010307 ผมคิดว่ายากพอๆ กันครับ

Mathophile 03 เมษายน 2007 17:20

อ้างอิง:

ข้อความเดิมเขียนโดยคุณ nongtum (ข้อความที่ 9352)
14. $2549|n^{2545}-2541\ \Rightarrow\ 2549|n^3(n^{2545}-2541)$ เนื่องจาก 2549 เป็นจำนวนเฉพาะ โดย Fermat จะได้
$n^{2548}-2541n^3\equiv1+8n^3\pmod{2549}$ เนื่องจาก $8n^3+1=(2n+1)(4n^2-2n+1)$ และ $4n^2-2n+1=2549$ ไม่มีคำตอบเป็นจำนวนเต็ม ดังนั้นค่า n ที่น้อยที่สุดคือ (2549-1)/2=1274

ขออนุญาตขุดของเก่าขึ้นมาถามนะครับ คืออ่านแล้วไม่เข้าใจตรงที่บอกว่า $4n^2-2n+1=2549$ ไม่มีคำตอบเป็นจำนวนเต็ม แล้วก็สรุปเลยว่าค่า n ที่น้อยที่สุดมาจากอีกวงเล็บนึง
ผมคิดว่า $4n^2-2n+1$ ไม่จำเป็นต้องเป็น 2549 น่ะครับ เป็นพหุคูณของ 2549 ก็ได้ เพระผมทำได้ว่า
$(2n-1)(4n^2-2n+1)\equiv 0\pmod{2549}$
ก็เลยแบ่งกรณีเป็น $2n-1\equiv 0\pmod{2549}$ หรือ $4n^2-2n+1\equiv 0\pmod{2549}$
กรณีแรกไม่มีปัญหา ผมเลยคิดว่าน่าจะพิสูจน์ให้ได้ว่า ไม่มีจำนวนนับใดตั้งแต่ $1-1273$ ที่ทำให้ $4n^2-2n+1\equiv 0\pmod{2549}$ ถึงจะได้ว่าค่าน้อยสุดคือ $1274$ อ่ะครับ หรือว่ามันเป็นอย่างไรกันแน่...? :confused:

warut 03 เมษายน 2007 18:47

ผมเห็นด้วยกับที่คุณ Mathophile พูดมาทั้งหมดครับ ข้อนี้อาจต้องใช้ความรู้เกี่ยวกับ quadratic residue หรือ อาจต้องใช้ถึง quadratic reciprocity law ถ้าไม่มีใครมาตอบ ว่างๆผมจะลองคิดดูครับ

R-Tummykung de Lamar 09 เมษายน 2007 02:11

ผมว่าตรงนี้ยังไม่ต้องใช้ทฤษฏีอะไรสูงมากนักครับ :happy:

$\large \text{สมมติให้ } 4n^2-2n+1\equiv 0\pmod{2549}$ $\mathbb{\qquad...(1)}$
ได้ $8n^3+1\equiv 0\pmod{2549}$
นั่นคือ $ (2n)^3\equiv-1\pmod{2549}$
จาก $ (2n)^{\phi(2549)=2548}\equiv-1\pmod{2549}$

ได้ว่า $-1\equiv (2n)^{1699(3)}=(2n)^{2(2548)+1}\equiv 2n\pmod{2549}$
นั่นคือ $2n+1 \equiv 0\pmod{2549} \qquad\mathbb{...(2)}$

ซึ่ง
$\gcd(2n+1,4n^2-2n+1)$
$=\gcd(2n+1,4n^2-2n+1-(2n-1)(2n+1))$
$=\gcd(2n+1,-2n+2)$
$=\gcd(2n+1,3)$
$\leq 3$

จาก $(1)$ และ $(2)$ ได้ $\gcd(2n+1,4n^2-2n+1) \geq 2549$ เกิดข้อขัดแย้ง

ดังนั้น $2549\not|4n^2-2n+1 \qquad \forall n\in \mathbb{Z}$

warut 09 เมษายน 2007 09:11

สวยงามมากครับ คุณ R-Tummykung de Lamar กลับมาคราวนี้เก่งขึ้นเยอะเลย :great:

แต่ว่า $(2n)^{2548}\equiv1\pmod{2549}$ ครับ และควรจะบอกด้วยว่าเราได้ตรงนี้มา เพราะเรารู้ว่า $2549\!\not|\;n$ เนื่องจาก $$n\equiv0\pmod{2549} \quad \Rightarrow \quad 8n^3+1 \not\equiv 0\pmod{2549}$$ และก็ไม่จำเป็นต้องใช้ Euler's $\phi$ function ด้วย เพราะตรงนี้เราใช้แค่ Fermat's Little Theorem ก็พอ ไม่จำเป็นต้องใช้ Euler-Fermat Theorem ครับ

ส่วนการพิสูจน์ว่า $$ 2n+1 \equiv0 \pmod{2549} \quad \Rightarrow \quad 4n^2-2n+1 \not\equiv0 \pmod{2549}$$ จะให้เหตุผลโดยใช้ความสัมพันธ์ $$ (4n^2-2n+1) - (2n-2)(2n+1) =3$$ ก็ได้ครับ

Mathophile 12 เมษายน 2007 16:12

ขอบคุณ คุณ R-Tummykung de Lamar และคุณ warut มากเลยครับ

nut123 13 มิถุนายน 2010 20:30

อ้างอิง:

ข้อความเดิมเขียนโดยคุณ warut (ข้อความที่ 17474)
ผมเห็นด้วยกับที่คุณ Mathophile พูดมาทั้งหมดครับ ข้อนี้อาจต้องใช้ความรู้เกี่ยวกับ quadratic residue หรือ อาจต้องใช้ถึง quadratic reciprocity law ถ้าไม่มีใครมาตอบ ว่างๆผมจะลองคิดดูครับ

อยากเห็นจังครับเผื่อจะมีประโยชน์บ้าง:)

์nat 14 มิถุนายน 2010 21:50

อยากได้ของศูนย์ มช มีบ้างมั้ยๆ


เวลาที่แสดงทั้งหมด เป็นเวลาที่ประเทศไทย (GMT +7) ขณะนี้เป็นเวลา 00:06

Powered by vBulletin® Copyright ©2000 - 2024, Jelsoft Enterprises Ltd.
Modified by Jetsada Karnpracha